整值函数的一个问题

问题描述

\(a=2+\sqrt{3}\), 有 \(n\in \mathbb{N}^{+}\), 求证明或推翻

\[4\lfloor an\rfloor=n+\lfloor a\lfloor an\rfloor\rfloor \]

我的想法

我感觉这个是对的。

展开:(定义 \(\{x\}=x-\lfloor x\rfloor\))

\(4an-4\{an\}=n+a\lfloor an\rfloor-\{a\lfloor an\rfloor\}\)

\(\therefore 4an-4\{an\}=n+a^{2}n-a\{an\}-\{a\lfloor an\rfloor\}\)

\(\therefore (4a-(a^{2}+1))n-4\{an\}=-a\{an\}-\{a\lfloor an\rfloor\}\)

\(\because 4a-(a^{2}+1)=4\times(2+\sqrt{3})-(2+\sqrt{3})^{2}-1=0\)

\(\therefore 4\{an\}=a\{an\}+\{a\lfloor an\rfloor\}\)

\(\because \{an\}=\{2\times n+\sqrt{3}\times n\}=\{n\sqrt{3}\}\)

\(\therefore 4\{n\sqrt{3}\}=a\{n\sqrt{3}\}+\{a\lfloor an\rfloor\}\)

\(\alpha=\{n\sqrt{3}\}\)

\(\therefore 4\alpha=a\alpha+\{a\lfloor an\rfloor\}\)

\(\therefore (4-a)\alpha=\{a\lfloor an\rfloor\}\)

\(\therefore (2-\sqrt{3})\alpha=\{a^2n-a\{an\}\}\)

\(\therefore (2-\sqrt{3})\alpha=\{a^2n-2\alpha-\alpha\sqrt{3}\}\)

然后就推不下去了,用同余试了试,好像不太可以。

解决关键

\[\begin{equation} \sqrt{3}n=n+k+d \label{split} \end{equation} \]

其中 \(k\in\mathbb{N^{+}},k\not=0,0\le d<1\)
\(k\)\(n(\sqrt{3}-1)\) 的整数部分,即 \(d\)\(n(\sqrt{3}-1)\) 的小数部分。

解答

\( \begin{align} 3n &=\sqrt{3}\times(\sqrt{3} \times n)\notag\\ &=\sqrt{3}\times(n+k+d)\notag && \text{代入}\eqref{split} \\ &=\sqrt{3}\times n+\sqrt{3}\times k+\sqrt{3}\times d\notag \end{align} \)

\(\sqrt{3}\times k+\sqrt{3}\times d\) 移到等式左边,得到:

\[\begin{equation} \sqrt{3}\times k=3n-\sqrt{3}\times n-\sqrt{3}\times d \label{sqrtkv} \end{equation} \]

展开左边:

\( \begin{align} 4\lfloor an\rfloor &=4\lfloor(2+\sqrt{3})n\rfloor\notag && \text{代入}a \\ &=4\lfloor2n+\sqrt{3}n\rfloor\notag\\ &=8n+4\lfloor\sqrt{3}n\rfloor\notag && \text{由} 8n\in \mathbb{Z} \\ &=8n+4\lfloor n+k+d\rfloor\notag && \text{代入}\eqref{split} \\ &=12n+4k+4\lfloor d\rfloor\notag \\ &=12n+4k\label{eq_left} \\ \end{align} \)

展开右边:

\( \begin{align} n+\lfloor a\times\lfloor a\times n\rfloor\rfloor &=n+\lfloor(2+\sqrt{3})\lfloor(2+\sqrt{3})n\rfloor\rfloor\notag\\ &=n+\lfloor(2+\sqrt{3})(2n+\lfloor\sqrt{3}n\rfloor)\rfloor\notag\\ &=n+\lfloor4n+2n\sqrt{3}+(2+\sqrt{3})\lfloor\sqrt{3}n\rfloor\rfloor\notag\\ &=5n+\lfloor2n\sqrt{3}+2\lfloor\sqrt{3}n\rfloor+\sqrt{3}\lfloor\sqrt{3}n\rfloor\rfloor\notag\\ &=5n+\lfloor2n\sqrt{3}+2(n+k)+\sqrt{3}(n+k)\rfloor\notag && \text{代入}\eqref{split}\\ &=7n+2k+\lfloor3n\sqrt{3}+\sqrt{3}k\rfloor\notag \\ &=7n+2k+\lfloor3n+3k+3d+\sqrt{3}k\rfloor\notag && \text{代入}\eqref{split}\\ &=10n+5k+\lfloor3d+\sqrt{3}k\rfloor\notag \\ &=10n+5k+\lfloor3d+(3n-\sqrt{3}n-\sqrt{3}d)\rfloor\notag && \text{代入}\eqref{sqrtkv}\\ &=13n+5k-\lfloor\sqrt{3}n\rfloor\notag \\ &=13n+5k-\lfloor n+k+d\rfloor\notag && \text{代入}\eqref{split} \\ &=12n+4k-\lfloor d\rfloor\notag \\ &=12n+4k\label{eq_right} \\ \end{align} \)

容易看出 \(\eqref{eq_left}=\eqref{eq_right}\)\(\therefore 4\times\lfloor a\times n\rfloor=n+\lfloor a\times\lfloor a\times n\rfloor\rfloor\) 证毕。

拓展:唯一性

有一数 \(a\in\mathbb{R^+}\), 使得对于 \(\forall n\in \mathbb{N^+}\), 存在

\[4\lfloor an\rfloor=n+\lfloor a\lfloor an\rfloor\rfloor \]

\(a\) 的值。

解答

\(\because a-1\le \lfloor a\rfloor\le a\)\(\therefore \begin{cases} 4a(n-1)\le a\lfloor an\rfloor=n+\lfloor a\lfloor an\rfloor\rfloor\le n+a(\lfloor an\rfloor-1) \\ 4an\ge a\lfloor an\rfloor=n+\lfloor a\lfloor an\rfloor\rfloor\ge n+a\lfloor an\rfloor \\ \end{cases}\)

  • 0
    点赞
  • 0
    收藏
    觉得还不错? 一键收藏
  • 0
    评论
评论
添加红包

请填写红包祝福语或标题

红包个数最小为10个

红包金额最低5元

当前余额3.43前往充值 >
需支付:10.00
成就一亿技术人!
领取后你会自动成为博主和红包主的粉丝 规则
hope_wisdom
发出的红包
实付
使用余额支付
点击重新获取
扫码支付
钱包余额 0

抵扣说明:

1.余额是钱包充值的虚拟货币,按照1:1的比例进行支付金额的抵扣。
2.余额无法直接购买下载,可以购买VIP、付费专栏及课程。

余额充值